LSAT and Law School Admissions Forum

Get expert LSAT preparation and law school admissions advice from PowerScore Test Preparation.

User avatar
 Dave Killoran
PowerScore Staff
  • PowerScore Staff
  • Posts: 5849
  • Joined: Mar 25, 2011
|
#72720
This game is also discussed in our Podcast, at the 48:04 mark: LSAT Podcast Episode 37: The November 2019 LSAT Logic Games Section

Complete Question Explanation

The correct answer choice is (D).

Answer choice (A): As discussed in the setup, both France and India cannot be visited by three separate representatives, and must instead be visited by exactly two representatives each.

Answer choice (B): As discussed in the setup, both France and India cannot be visited by three separate representatives, and must instead be visited by exactly two representatives each.

Answer choice (C): Because of the placement of V and the YW block in the Ghana row, this answer cannot occur.

Answer choice (D): This is the correct answer choice.

Answer choice (E): Because V must visit India twice, this answer cannot occur.
User avatar
 khalifeh19
  • Posts: 1
  • Joined: Jan 04, 2021
|
#82955
Can you diagram an example of how D would look please? I cant seem to prove it on my own.
User avatar
 KelseyWoods
PowerScore Staff
  • PowerScore Staff
  • Posts: 1079
  • Joined: Jun 26, 2013
|
#83018
Hi khalifeh19!

One possible solution that would prove answer choice (D) is possible would be:

F: W Z Z Z
G: Y W V W
I: V Y Y V

Hope this helps!

Best,
Kelsey
User avatar
 rocketman16
  • Posts: 13
  • Joined: Jul 26, 2021
|
#89652
For future reference to whoever takes this PT down the road, I figured that this was a pretty easy question to get just by process of elimination:

A: We know that at least 2 reps have to visit India, and anyone who visits India cannot visit France. Eliminate.

B: Same rationale as option A. Eliminate.

C: We know that we need to have V and the YW block in Ghana. V can only appear in Ghana once, so obviously that one cannot appear twice in a row. Z also obviously cannot appear twice in Ghana at all. Because the placement of V limits the placement of the YW block, if V is in month 3 for Ghana, the YW block must occupy months 1 and 2, and you cannot add another Y or W in succession. If you place V in month 2, you can flip the YW block placement and see the same problem. Eliminate.

E: Two of the India slots are automatically occupied by V, so no other rep could possibly appear 3 times at all. Eliminate.
User avatar
 JoshuaDEL
  • Posts: 15
  • Joined: Apr 25, 2021
|
#89695
KelseyWoods wrote: Wed Jan 06, 2021 11:04 pm Hi khalifeh19!

One possible solution that would prove answer choice (D) is possible would be:

F: W Z Z Z
G: Y W V W
I: V Y Y V

Hope this helps!

Best,
Kelsey
Hello,

The third rule says that "Y must visit G in a month immediately preceding a month in which W visits G"

I was stuck with this question because I thought the example you just gave is not possible since the last W in G must be also preceded by Y forcing it to be G: YWYW which is not possible. I'm assuming from this explanation that the rule meant only one instance but how are we to know that (maybe "a" month)? What part of the rule allows us to eliminate that possibility?

Thanks!
 Adam Tyson
PowerScore Staff
  • PowerScore Staff
  • Posts: 5153
  • Joined: Apr 14, 2011
|
#89704
That's exactly right, Joshua! The use of "a month" means that the YW block must happen at least once (and in fact it cannot happen more than once). That's different from "Y must visit G in the month immediately preceding any month in which W visits G."

In short, this rule means that there must be a YW block in G, but does not preclude W from showing up a second time without Y.
User avatar
 JoshuaDEL
  • Posts: 15
  • Joined: Apr 25, 2021
|
#89732
Adam Tyson wrote: Sun Aug 15, 2021 11:11 am That's exactly right, Joshua! The use of "a month" means that the YW block must happen at least once (and in fact it cannot happen more than once). That's different from "Y must visit G in the month immediately preceding any month in which W visits G."

In short, this rule means that there must be a YW block in G, but does not preclude W from showing up a second time without Y.
Makes sense thanks!
 dimi.wassef@yahoo.com
  • Posts: 34
  • Joined: Aug 26, 2021
|
#92486
Why is A wrong? I can't seem to understand why YWZ couldn't all go in France.
 Robert Carroll
PowerScore Staff
  • PowerScore Staff
  • Posts: 1783
  • Joined: Dec 06, 2013
|
#92516
dimi.wassef,

This initial post in this thread says, about answer choice (A), "As discussed in the setup..." The setup thread is here: viewtopic.php?f=1337&t=31743 The following quote from that should answer your question fully:

Each site cannot have just one representative visit, because there are four months, and each representative only makes three visits. So, just from the structural setup of the game each site would have to have at least two representatives visit.

There also cannot be three different representatives to visit France or India because if one of those sites had three representatives visit, then the other site could not have those three representatives visit, meaning only one representative would be available, and we've already established that is impossible. Of course, if three representatives is impossible, four is also impossible.

Thus, we can establish that France and India each have exactly two representatives visit the site, and due to the language in the fifth rule, those must each be two different pairs of reps.
Robert Carroll

Get the most out of your LSAT Prep Plus subscription.

Analyze and track your performance with our Testing and Analytics Package.